Why are Bell's inequalities violated?

In summary, the conversation discusses the reasons for the violation of Bell's inequalities, which are based on the measurement of non-commuting quantum observables. It is noted that the experiments themselves show that the underlying "hidden variables" may interact with the measuring device in an unknown physical way, leading to a change in values. This challenges the assumption that the measurement of one observable does not affect the value of another, which is the basis of Bell's inequality. The conversation also mentions a paper by A. Peres, which further explores this concept and concludes that there is no way to account for the predictions of quantum mechanics using hypothetical unobserved experiments.
  • #36
DrChinese said:
That does not make sense, kith. It doesn't matter whether a theory is classical or not! That is a completely arbitrary designation.

The fact is, there is no theory - now or ever - which explains how the observer's past has anything whatsoever to do with ANY experiment. That includes QM. It is just a blind ad hoc hypothesis thrown out by a few people. So you cannot explain WHY it should apply to entanglement more (or less) than the age of the universe or measurements of c or anything else.
I agree with this. The assumption of superdeterminism is useless for understanding why Bell's lhv formulation is inappropriate (ie., why it produces incorrect predictions) for modeling Bell tests.
 
Physics news on Phys.org
  • #37
DrChinese said:
[..]
In my book, you pick and choose what evidence you accept, in order to be consistent with your pre-ordained conclusion.
Why would you do that? Or was it meant as a personal attack?
 
  • #38
nanosiborg said:
I agree with this. The assumption of superdeterminism is useless for understanding why Bell's lhv formulation is inappropriate (ie., why it produces incorrect predictions) for modeling Bell tests.
Do you think the observer obeys the laws of nature, i.e. can be considered as a physical system?
 
  • #39
kith said:
Do you think the observer obeys the laws of nature, i.e. can be considered as a physical system?
Sure. But the OP is concerned with why Bell's inequalities are violated. How will an ad hoc metaphysical assumption, such as superdeterminism, inform regarding that?
 
  • #40
nanosiborg said:
Sure.
Do you agree that in a deterministic theory, the behaviour of a physical system is determined by its past or current state? Do you agree that in such a theory, the behaviour of the observer is determined by its past or current state if we treat him as a physical system?

nanosiborg said:
But the OP is concerned with why Bell's inequalities are violated. How will an ad hoc metaphysical assumption, such as superdeterminism, inform regarding that?
Bell's theorem makes the assumption that the probability distributions for Alice and Bob are independent. If you don't make this assumption, you can't derive the inequality in the first place. So discussing this assumption seems relevant to me. Personally, I haven't completely wrapped my mind around superdeterminism and want to understand the arguments more deeply.
 
  • #41
nanosiborg said:
Assuming either superdeterminism or nonlocality is not informative.The answer is in the realm of anti-realism, which has to do with modeling restrictions.
Just to be clear, when you are using the term "anti-realism", do you mean: no pre-existing properties (non-counterfactuals). I'm asking because this stuff is a bit confusing as there are problems even with what is meant by "realism" also. For instance Wood and Spekkens write:
It has always been rather unclear what precisely is meant by "realism". Norsen has considered various philosophical notions of realism and concluded that none seem to have the feature that one could hope to save locality by abandoning them. For instance, if realism is taken to be a commitment to the existence of an external world, then the notion of locality-that every causal influence between physical systeems propagates subluminally already presupposes realism.
The lesson of causal discovery algorithms for quantum correlations: Causal explanations of Bell-inequality violations require fine-tuning
http://arxiv.org/pdf/1208.4119v1.pdf

Perhaps, Maccone's definition of "realism" seems as one of the more clearer ones:
Let us define “counterfactual” a theory whose experiments uncover properties that are pre-existing. In other words, in a counterfactual theory it is meaningful to assign a property to a system (e.g. the position of an electron) independently of whether the measurement of such property is carried out. Sometime this counterfactual definiteness property is also called “realism”, but it is best to avoid such philosophically laden term to avoid misconceptions. Bell’s theorem can be phrased as “quantum mechanics cannot be both local and counterfactual”. A logically equivalent way of stating it is “quantum mechanics is either non-local or non-counterfactual”.
Simplest proof of Bell’s inequality
http://arxiv.org/pdf/1212.5214v1.pdf

But I always have trouble understanding this. If something is not pre-existing, would not the Wood and Spekkens argument above hold?
 
  • #42
kith said:
Usually, we have a system S and an observer O measuring some observable of the system. As soon as we consider the combined system S+O as a physical system (which may be observed by another observer O'), we acknowledge that the current state of S and O influences the evolution of the combined system. This evolution may of course include interactions between S and O. What's wrong with this kind of thinking?

It can apply to any theory equally! It certainly applies to special relativity, that is it's very reason to exist is to account for observer reference frames. There is absolutely no reason to suspect that there is anything special in that regards about QM. And not the slightest evidence to suspect that the observer's PAST influences the outcome (only the observer's choice of measurement setting).
 
  • #43
harrylin said:
Why would you do that? Or was it meant as a personal attack?

I don't attack people, sorry if anything I said implied otherwise.

However, I think it is clear that you value a specific view that is usually excluded by Bell+experiment. You are certainly not the only one. However, it is very difficult to discuss the subject meaningfully when you assume that which you seek to prove.

Of course, for all I know you might say the same thing about my viewpoint. :smile:
 
  • #44
nanosiborg said:
Unlikely because it's an assumption without evidence. Convenient because, using Bell's formulation, assuming nonlocality allows that the results at one end depend on the analyzer settings at the other end. Too convenient for my taste. Maybe not yours and others. But at this point it is just a matter of taste.

I think that something other than nonlocality will eventually answer the thread question.

Suppose there are two sorts of time, one in which the entangled particles in Aspect's experiment both change state at the same time and one (the usual one) in which their change of state should be separated by the amount of time it takes for the effect to travel between the two events at the speed of light, but is not observed to do so, hence our problem. Would that help?
 
  • #45
kith said:
Do you agree that in a deterministic theory, the behaviour of a physical system is determined by its past or current state? Do you agree that in such a theory, the behaviour of the observer is determined by its past or current state if we treat him as a physical system?
I agree with what DrChinese said. It's irrelevant.

kith said:
Bell's theorem makes the assumption that the probability distributions for Alice and Bob are independent. If you don't make this assumption, you can't derive the inequality in the first place. So discussing this assumption seems relevant to me. Personally, I haven't completely wrapped my mind around superdeterminism and want to understand the arguments more deeply.
I believe that superdeterminism is clutching at straws, and that it will not help us to understand the incompatibility between the lhv formalism and experiment. On the other hand, Bell's formulation of the independence assumption (his locality condition) is relevant, and some people (eg., Jarrett) think that a component of it (which, re Jarrett's analysis, doesn't necessarily inform regarding locality/nonocality in nature) might be the effective cause of BI violation.

Here's another way to approach the OP question. What is it about a basic Bell lhv model that produces a linear correlation (which is incompatible with the nonlinear one produced by qm and experiment) between θ and rate of coincidental detection?


bohm2 said:
Just to be clear, when you are using the term "anti-realism", do you mean: no pre-existing properties (non-counterfactuals). I'm asking because this stuff is a bit confusing as there are problems even with what is meant by "realism" also.
I should have said that I think the answer has to do with some aspect of the realism of Bell's formulation, which includes the separable functions A and B (which describe individual detection) expressed in terms of λ, and the expression of the independence (locality) assumption in terms of the functions A and B -- with the result of the Bell lhv program being against realism or "anti-realism" in that only nonrealistic models of quantum entanglement, such as in standard qm or MWI, are allowed (unless you assume ftl or instantaneous action at a distance, such as in dBB).

bohm2 said:
But I always have trouble understanding this. If something is not pre-existing, would not the Wood and Spekkens argument above hold?
I don't know. I don't understand the Wood and Spekkens article. Maybe you can explain it?

Dan Fitzgibbon said:
Suppose there are two sorts of time, one in which the entangled particles in Aspect's experiment both change state at the same time and one (the usual one) in which their change of state should be separated by the amount of time it takes for the effect to travel between the two events at the speed of light, but is not observed to do so, hence our problem. Would that help?
I don't see how it would. There's no particularly compelling reason to posit effects traveling between the two events. Paired detection events aren't correlated with each other except when the analyzer settings are aligned, in which case a local common cause explanation suffices.
 
  • #46
nanosiborg said:
...only nonrealistic models of quantum entanglement, such as in standard qm or MWI, are allowed.
MWI is a "realistic" interpretation.
 
  • #47
bohm2 said:
MWI is a "realistic" interpretation.
Ok, then change ...
"with the result of the Bell lhv program being against realism or 'anti-realism' in that only nonrealistic models of quantum entanglement, such as in standard qm or MWI, are allowed (unless you assume ftl or instantaneous action at a distance, such as in dBB)"
... to ...
"with the result of the Bell lhv program being against realism or 'anti-realism' in that only models of quantum entanglement which don't employ hidden variables, such as in standard qm or MWI, are allowed (unless you assume ftl or instantaneous action at a distance, such as in dBB)".

So my answer to your question ...
"Just to be clear, when you are using the term "anti-realism", do you mean: no pre-existing properties ... ?"
... would be that when I'm using the term "anti-realism", I mean no hidden variables.
 
  • #48
bohm2 said:
[..]
Simplest proof of Bell’s inequality
http://arxiv.org/pdf/1212.5214v1.pdf

But I always have trouble understanding this. If something is not pre-existing, would not the Wood and Spekkens argument above hold?
Thanks for that link - regretfully they handle the assumption that it "is meaningful to assign a property to a system (e.g. the position of an electron) independently of whether the measurement of such property is carried out." However I see no reason to believe that this must always be applicable, and I thought that also Bell did not impose such a requirement to "realism" (or did I overlook it somewhere?).
 
Last edited:
  • #49
harrylin said:
I thought that also Bell did not impose such a requirement to "realism" (or did I overlook it somewhere?).

Yes, sadly Bell does not highlight this point. I guess he thought it was obvious. It is the spot after his (14) where he says, "It follows that c is another unit vector". The idea is that a, b and c are simultaneously real (i.e. elements of reality). So this is the spot where that assumption takes place, and without it he would not be able to continue to derive his conclusion.
 
  • #50
DrChinese said:
Yes, sadly Bell does not highlight this point. I guess he thought it was obvious. It is the spot after his (14) where he says, "It follows that c is another unit vector". The idea is that a, b and c are simultaneously real (i.e. elements of reality). So this is the spot where that assumption takes place, and without it he would not be able to continue to derive his conclusion.
:bugeye: :uhh: I never realized that, and will have to check it out - thanks! :smile:
 
  • #51
harrylin said:
:bugeye: :uhh: I never realized that, and will have to check it out - thanks! :smile:

Yes, and note that this is the very first equation with a, b and c in it. Without all 3, of course, you don't have realism expressed as a testable assumption.
 
  • #52
nanosiborg said:
I don't know. I don't understand the Wood and Spekkens article. Maybe you can explain it?
If one argues that something is local, realism is implied as above posts, I think. Analogously, if non-realism, then the issue of locality vs non-locality is kind of pointless since there's no ontological issues. I mean what ontological difference would there be between local vs non-local non-realism? Anyway, that's how I understood it. I think Gisin argues similarily here:
What is surprising is that so many good physicists interpret the violation of Bell’s inequality as an argument against realism. Apparently their hope is to thus save locality, though I have no idea what locality of a non-real world could mean? It might be interesting to remember that no physicist before the advent of relativity interpreted the instantaneous action at a distance of Newton’s gravity as a sign of non-realism (although Newton’s nonlocality is even more radical than quantum nonlocality, as it allowed instantaneous signaling).
Is realism compatible with true randomness?
http://arxiv.org/pdf/1012.2536v1.pdf
 
Last edited:
  • #53
DrChinese said:
Yes, and note that this is the very first equation with a, b and c in it. Without all 3, of course, you don't have realism expressed as a testable assumption.
I certainly recall that point, which was the subject of many discussions... I just never interpreted it in the sense as expressed in that paper!

Their clarification ("assign a property to a system (e.g. the position of an electron)") plus your explanation allows me to make sense of Neumaier's claim (which he did not prove and with which you probably disagree). He holds that:
"all proofs of Bell type results [..] become invalid when particles have a temporal and spatial extension, with an internal structure that is modified when interacting in a beam splitter."
- http://arnold-neumaier.at/ms/lightslides.pdf

So, he suggests that that is why Bell's inequalities are violated.
 
  • #54
harrylin said:
I certainly recall that point, which was the subject of many discussions... I just never interpreted it in the sense as expressed in that paper!

Their clarification ("assign a property to a system (e.g. the position of an electron)") plus your explanation allows me to make sense of Neumaier's claim (which he did not prove and with which you probably disagree). He holds that:
"all proofs of Bell type results [..] become invalid when particles have a temporal and spatial extension, with an internal structure that is modified when interacting in a beam splitter."
- http://arnold-neumaier.at/ms/lightslides.pdf

So, he suggests that that is why Bell's inequalities are violated.

He makes several statements like this that are difficult for me to place in a suitable context. Photons have both temporal and spatial extension in my view. Not sure if they do to a local realist though. And I do not follow his reasoning on how that connects to Bell. So I guess I would say that I disagree. If you are enforcing strict Einsteinian locality as is done in Weihs et al (1998), the spatial extent of a photon is already accounted for.
 
  • #55
DrChinese said:
[..] I do not follow his reasoning on how that connects to Bell. [..]
Neither did I follow his reasoning until you explained it to me! Now I guess that I can follow it, for the first time.

Assuming that light is a wave and not a collection of photon particles, then the photon positions are undefined or non-existent until they are measured - and the same for some other properties. If so, then Bell's theorem may not apply because the existence of such unmeasured properties is required for that theorem - is that correct? It should be, following our posts #49 and #50 here above.
 
  • #56
harrylin said:
Assuming that light is a wave and not a collection of photon particles, then the photon positions are undefined or non-existent until they are measured - and the same for some other properties. If so, then Bell's theorem may not apply because the existence of such unmeasured properties is required for that theorem - is that correct?

I keep saying that you have it backwards. QM does not assert photons have well-defined values for non-commuting operators - realists do!

EPR asserts that there is an element of reality IF Alice can predict Bob's outcome with certainty. Because this is experimentally demonstrated, you must accept EPR's key challenge. Which is that if QM is complete (read accurate in this instance), the reality of Bob's measurement is a function of Alice's choice of what to observe. Therefore if QM predicts correctly, we live in an observer dependent universe. This is directly from EPR. It does require the assumption of SIMULTANEOUS elements of reality (anything else is an unreasonable definition of reality, they say) and the assumption that there is no spooky action at a distance.

Bell simply takes those one step forward in his proof. So sure, Bell doesn't "apply" in the sense that one of the local realists' (and EPR's) 2 assumptions is invalid (much as you say: "the existence of such unmeasured properties"). But that is simply agreeing with Bell, disagreeing with EPR and denying local realism in one breath.
 
  • #57
harrylin said:
I certainly recall that point, which was the subject of many discussions... I just never interpreted it in the sense as expressed in that paper!

Their clarification ("assign a property to a system (e.g. the position of an electron)") plus your explanation allows me to make sense of Neumaier's claim (which he did not prove and with which you probably disagree). He holds that:
"all proofs of Bell type results [..] become invalid when particles have a temporal and spatial extension, with an internal structure that is modified when interacting in a beam splitter."
- http://arnold-neumaier.at/ms/lightslides.pdf

So, he suggests that that is why Bell's inequalities are violated.

I don't know, maybe A. Neumaier has revised his text since you looked at it, but I find a slightly different phrase there: "All proofs of Bell type results (including the present argument) become invalid when "particles" have a temporal and spatial extension over the whole experimental domain, with an internal structure that is modified when interacting in a beam splitter."

These extra words ("over the whole experimental domain") make me wonder if what he had in mind might be pretty much the same as the locality loophole.
 
  • #58
akhmeteli said:
I don't know, maybe A. Neumaier has revised his text since you looked at it, but I find a slightly different phrase there: "All proofs of Bell type results (including the present argument) become invalid when "particles" have a temporal and spatial extension over the whole experimental domain, with an internal structure that is modified when interacting in a beam splitter."

These extra words ("over the whole experimental domain") make me wonder if what he had in mind might be pretty much the same as the locality loophole.

I saw that too, couldn't figure out what he meant. Does he mean: non-local? Obviously the locality loophole itself was closed a while back.
 
  • #59
DrChinese said:
I saw that too, couldn't figure out what he meant. Does he mean: non-local? Obviously the locality loophole itself was closed a while back.

Only separately:-)
 
  • #60
akhmeteli said:
Only separately:-)

I would expect no less! :smile:
 
  • #61
In a previous post i wrote down the CHSH inequality that any hidden variable model satisfies
JK423 said:
[itex]S_j = A_j\left( {{a_1}} \right)B_j\left( {{b_1}} \right) + A_j\left( {{a_1}} \right)B_j\left( {{b_2}} \right) + A_j\left( {{a_2}} \right)B_j\left( {{b_1}} \right) - A_j\left( {{a_2}} \right)B_j\left( {{b_2}} \right)[/itex],
where [itex]A_j\left( {{a_i}} \right) = \pm 1[/itex] and [itex]B_j\left( {{b_i}} \right) = \pm 1[/itex], and j denoting a particular photon pair,
is always [itex]{S_j} = \pm 2[/itex], for any measurement result A and B.
When we take the mean value over all photon pairs, [itex]\,\left\langle S \right\rangle = \frac{1}{N}\sum\limits_{i = 1}^N {{S_j}} [/itex] we find it to be bounded, i.e.
[itex] - 2 \le \,\left\langle S \right\rangle \le 2[/itex].

I have understood quite well why every photon pair should satisfy this inequality (if local realism holds). Consequently, the mean value [itex]\left\langle S \right\rangle [/itex], over all photon pairs, should satisfy it as well. However, this quantity is unmeasurable in a real experiment, due to the fact that it involves unmeasurable quantities in each run, e.g. the covariances in different angles.
Now my problem is to express the CHSH inequality in an equivalent way, so that it involves only measurable quantities, like the number of pairs detected anti-correlated when measured in the angle [itex]\theta_1[/itex], and consequently will be applicable in a real experiment. But i want to do this completely equivalently! For example, the fact that in the quoted post i have written down the inequality satisfied by the mean value of S, i.e.
[itex] - 2 \le \,\left\langle S \right\rangle \le 2[/itex],
it doesn't mean that this inequality would also be satisfied by the mean value of the measurements! The mean value of S (including all unmeasurable quantities), that satisfies the above inequality, and the mean value of S which includes only experimental mean values (that don't include the unmeasurable quantities) are not equivalent! They are not equivalent because, in the first case the mean value of each quantity [itex]\left\langle {A\left( {{a_1}} \right)B\left( {{b_2}} \right)} \right\rangle [/itex] involves all photon pairs in the experiment, while in a real experiment the corresponding mean value would include only 1/4 of the total photon pairs (if we suppose that the four angles are chosen with the same probability), since with each photon pair we can measure only one of the 4 observables. The two quantities are totally different.

So, what i am looking for is an equivalent but measurable expression. Does anyone have any idea of how to do this?
Note: The above CHSH inequality holds for every initial preparation of the photons, so in the proposed derivation we should not include any condition on the initial state in order to have a more general result.
 
Last edited:
  • #62
JK423 said:
In a previous post i wrote down the CHSH inequality that any hidden variable model satisfiesI have understood quite well why every photon pair should satisfy this inequality (if local realism holds). Consequently, the mean value [itex]\left\langle S \right\rangle [/itex], over all photon pairs, should satisfy it as well. However, this quantity is unmeasurable in a real experiment, due to the fact that it involves unmeasurable quantities in each run, e.g. the covariances in different angles.
Now my problem is to express the CHSH inequality in an equivalent way, so that it involves only measurable quantities, like the number of pairs detected anti-correlated when measured in the angle [itex]\theta_1[/itex], and consequently will be applicable in a real experiment. But i want to do this completely equivalently! For example, the fact that in the quoted post i have written down the inequality satisfied by the mean value of S, i.e.
[itex] - 2 \le \,\left\langle S \right\rangle \le 2[/itex],
it doesn't mean that this inequality would also be satisfied by the mean value of the measurements! The mean value of S (including all unmeasurable quantities), that satisfies the above inequality, and the mean value of S which includes only experimental mean values (that don't include the unmeasurable quantities) are not equivalent! They are not equivalent because, in the first case the mean value of each quantity [itex]\left\langle {A\left( {{a_1}} \right)B\left( {{b_2}} \right)} \right\rangle [/itex] involves all photon pairs in the experiment, while in a real experiment the corresponding mean value would include only 1/4 of the total photon pairs (if we suppose that the four angles are chosen with the same probability), since with each photon pair we can measure only one of the 4 observables. The two quantities are totally different.

So, what i am looking for is an equivalent but measurable expression. Does anyone have any idea of how to do this?
Note: The above CHSH inequality holds for every initial preparation of the photons, so in the proposed derivation we should not include any condition on the initial state in order to have a more general result.

This is a point that is so confusing, I would say most folks reason do not understand at all. S is a derived formula, and to a certain extent, an arbitrary expression. There is absolutely NO need to be able to measure this in a single experiment.

What you are really attempting to do is to verify the QM prediction of cos^2(theta) for matches. That is experimentally verifiable. If QM predicts accurately (with cos^2), then Bell tells us that LR is violated. There is then no need to have the CHSH inequality. And if you look at a lot of the Bell tests, they graph the QM predicted value against the experimental results to show this. (The LR function would need to be a straight line, in contrast.)

However, it is possible to look at the graphed results and say, "hmmm, maybe a straight line as close to the observed results as the QM prediction". This is where CHSH comes in. It is a concrete way to determine that LR is rejected while QM is confirmed.

So specifically: the coincidence prediction for 60 degrees for QM is .25 and for LR is .33 or higher. All you have to do is measure that, it is directly measurable exactly as you hope! Then you see that the measured value is quite close to .25 and far away from .33 (by perhaps 30+ standard deviations). And you rule out LR because its prediction is flat out wrong.
 
  • #63
Thank you both, DrChinese and Gordon Watson, for your feedback.
Let me restate where my doubts are specifically located, so that my point will become more clear and you will be able to give me more "targeted" help. First, let me repeat the necessary formulas:
JK423 said:
[itex]S_j = A_j\left( {{a_1}} \right)B_j\left( {{b_1}} \right) + A_j\left( {{a_1}} \right)B_j\left( {{b_2}} \right) + A_j\left( {{a_2}} \right)B_j\left( {{b_1}} \right) - A_j\left( {{a_2}} \right)B_j\left( {{b_2}} \right)[/itex],
where [itex]A_j\left( {{a_i}} \right) = \pm 1[/itex] and [itex]B_j\left( {{b_i}} \right) = \pm 1[/itex], and j denoting a particular photon pair,
is always [itex]{S_j} = \pm 2[/itex], for any measurement result A and B.
When we take the mean value over all photon pairs, [itex]\,\left\langle S \right\rangle = \frac{1}{N}\sum\limits_{i = 1}^N {{S_j}} [/itex] we find it to be bounded, i.e.
[itex] - 2 \le \,\left\langle S \right\rangle \le 2[/itex].
This quantity is bounded whatever the values of A and B for the photon pairs.

Now, the reason why the CHSH inequality holds is because [itex]S_j[/itex] is always [itex]S_j=±2[/itex] for every photon pair separately. And this is due to the locality assumption, i.e. that the outcome in Alice's side does not depend on what Bob measures, etc. Mathematically this assumption is expressed by the fact that [itex]{A_j}\left( {{a_1}} \right)[/itex] is the same in both [itex]{A_j}\left( {{a_1}} \right) \cdot {B_j}\left( {{\beta _1}} \right)[/itex] and [itex]{A_j}\left( {{a_1}} \right) \cdot {B_j}\left( {{\beta _2}} \right)[/itex], i.e. whether Bob measures [itex]\beta_1[/itex] or [itex]\beta_2[/itex] is irrelevant, the outcome [itex]{A_j}\left( {{a_1}} \right)[/itex] will be the same. The same reasoning applies to [itex]{A_j}\left( {{a_2}} \right)[/itex].
The important thing here is that this is always true because [itex]{A_j}\left( {{a_1}} \right)[/itex] corresponds to the same photon in these two expressions [itex]{A_j}\left( {{a_1}} \right) \cdot {B_j}\left( {{\beta _1}} \right)[/itex] and [itex]{A_j}\left( {{a_1}} \right) \cdot {B_j}\left( {{\beta _2}} \right)[/itex], so it cannot be different if locality is assumed.

Now, look what happens if you consider the measureable edition of [itex]\left\langle S \right\rangle[/itex], where each of the quantities [itex]\left\langle {A\left( {{a_i}} \right)B\left( {{b_j}} \right)} \right\rangle[/itex] are mean values over the measurements. For simplicity let me consider only one of the measured values (instead of the whole mean value) in order to make my point clear. So assume just one run:
[itex]\left\langle {A\left( {{a_1}} \right)B\left( {{b_1}} \right)} \right\rangle = {A_1}\left( {{a_1}} \right){B_1}\left( {{b_1}} \right)[/itex] , corresponding to the measured value of the photon pair "1",
[itex]\left\langle {A\left( {{a_1}} \right)B\left( {{b_2}} \right)} \right\rangle = {A_2}\left( {{a_1}} \right){B_2}\left( {{b_2}} \right)[/itex], corresponding to the measured value of the photon pair "2".
Now take the sum of these in order to form the first half part of the quantity S:
[itex]\left\langle {A\left( {{a_1}} \right)B\left( {{b_1}} \right)} \right\rangle + \left\langle {A\left( {{a_1}} \right)B\left( {{b_2}} \right)} \right\rangle = {A_1}\left( {{a_1}} \right){B_1}\left( {{b_1}} \right) + {A_2}\left( {{a_1}} \right){B_2}\left( {{b_2}} \right)[/itex]. (1)

I told you previously that CHSH holds because [itex]{A_j}\left( {{a_1}} \right)[/itex] has the same value in these two quantities, since it corresponds to the same photon. But now that we have considered the mean values over measurements, [itex]{A_1}\left( {{a_1}} \right)[/itex] and [itex]{A_2}\left( {{a_1}} \right)[/itex] are, generally, different since they correspond to different photons "1" and "2", and that way they could be mimicking non-locality, since it looks as if [itex]{A}\left( {{a_1}} \right)[/itex] depends on what Bob measures.
You can generalize (1) for N photon pairs and take a more appropriate mean value. The moral in this story is that the photons are different in each quantity, so there is no obvious reason why CHSH would not be violated.

I hope that i made my point clear..
I'm looking forward to your feedback!

Giannis
 
  • #64
bohm2 said:
If one argues that something is local, realism is implied as above posts, I think. Analogously, if non-realism, then the issue of locality vs non-locality is kind of pointless since there's no ontological issues. I mean what ontological difference would there be between local vs non-local non-realism? Anyway, that's how I understood it. I think Gisin argues similarily here:

Is realism compatible with true randomness?
http://arxiv.org/pdf/1012.2536v1.pdf
Thanks for helping me wade through this bohm2.
bohm2 said:
If one argues that something is local, realism is implied as above posts, I think.
What about, eg., QFT?

We're (as Bell was) concerned with lhv formalism as it relates to qm and quantum entanglement experiments ... and not with how lhv formalism relates to the ontology of the world.
Can we agree that realism, for our purposes, means the formal expression of hidden variables?

We see by dBB that hidden variables and therefore hv formalisms aren't ruled out. But lhv (at least Bell lhv) formalisms are. So, it seems to come down to something to do with the formal expression of locality in terms of hidden variables ... and I'm reminded again of Jarrett's (and similar) treatment(s) of this which say that BI violation might be ruling out Bell type lhv models without also ruling out the possibility that nature is exclusively local.
 
Last edited:
  • #65
DrChinese said:
[...] if QM is complete (read accurate in this instance), the reality of Bob's measurement is a function of Alice's choice of what to observe. [..]
It does require the assumption of SIMULTANEOUS elements of reality (anything else is an unreasonable definition of reality, they say) and the assumption that there is no spooky action at a distance.
As I just discovered and clarified, it's exactly the meaning of those "simultaneous elements of reality" that appears to be an unreasonable requirement if that means "counterfactual" in the sense as cited in post #49. So, I guess that we now discovered in two parallel threads (and for different reasons) that it may be useful to focus more on Bell's "realist" criteria.
[..] But that is simply agreeing with Bell, disagreeing with EPR and denying local realism in one breath.[/b]
Once more: thanks for pointing out that Bell-realism is only a particular form of realism, different from that of Neumaier and myself. I will have to read again EPR to verify if their formulation of "realism" was as limited as Bell's.
akhmeteli said:
I don't know, maybe A. Neumaier has revised his text since you looked at it, but I find a slightly different phrase there: "All proofs of Bell type results (including the present argument) become invalid when "particles" have a temporal and spatial extension over the whole experimental domain, with an internal structure that is modified when interacting in a beam splitter."

These extra words ("over the whole experimental domain") make me wonder if what he had in mind might be pretty much the same as the locality loophole.
I did a copy-paste so that's puzzling... I forgot what is meant with "locality loophole", but I'm pretty sure that he refers to his interpretation of QFT [EDIT:"Photons are intrinsically nonlocal objects"] which I think differs from the kind of spatial extension that DrChinese has in mind.
 
Last edited:
  • #66
In this very basic table of the 23 possible cases of a spin 1/2 system with 3
axis settings, x,y,z ;

A _________ B
x y z ______ x y z
+++ _______ ---
++- _______ --+
+-+ _______ -+-
+-- _______ -++
-++ _______ +--
-+- _______ +-+
--+ _______ ++-
--- _______ +++

P(x+y+) < P(x+z+)+P(z+y+)
In the above inequality what are the exact counts that violate it ? And if the magnetic
field in the detector not only alters the spin on y-axis when detecting spin on x but also
alters the spin on the axis being measured, x , then how is this violation valid ?
 
Last edited:
  • #67
morrobay said:
In this very basic table of the 23 possible cases of a spin 1/2 system with 3
axis settings, x,y,z ;

A _________ B
x y z ______ x y z
+++ _______ ---
++- _______ --+
+-+ _______ -+-
+-- _______ -++
-++ _______ +--
-+- _______ +-+
--+ _______ ++-
--- _______ +++

P(x+y+) < P(x+z+)+P(z+y+)
In the above inequality what are the exact counts that violate it ? And if the magnetic
field in the detector not only alters the spin on y-axis when detecting spin on x but also
alters the spin on the axis being measured, x , then how is this violation valid ?

The cases you show assume a realistic/hidden variable perspective. OK, that is fine for a starting point for a Bell Inequality. Are you thinking that x, y and z are 3 perpendicular spatial axes (not clear to me from the example) ? Because if so you can't get a Bell Inequality from those. Instead, you need something like:

x=0
y=135
z=90

Assuming this is fine with you, and these angles are on the same plane:

xz= theta of 90 degrees
yz= theta of 45 degrees
xy= theta of 135 degrees

The quantum mechanical prediction for Matches (M) when anti-correlated is: 1 - cos^2(theta/2) or simply sin^2(theta/2). Your realistic requirement is M(xy) < M(xz) + M(yz) which is the same as saying: 0 < M(xz) + M(yz) - M(xy). Substituting the right hand side, you get something like:

M(xz)* + M(yz) - M(xy) =

sin^2(90/2) + sin^2(45/2) - sin^2(135/2) =

.5 + .1465 - .8535 =

-.2070**

Oops, this was supposed to be greater than zero per your realism requirement! So the realism requirement is flat out inconsistent with the predictions of QM. So here are the specific values that lead to a violation.

*There was a minor issue in your formula that became immaterial because of the angle settings I selected.
** And this is reduced by half to -.1035 if we only look at the ++ match cases, not that it really matters. The QM prediction is still less than zero and realism requires it be non-negative.
 
  • #68
DrChinese said:
The quantum mechanical prediction for Matches (M) when anti-correlated is: 1 - cos^2(theta/2) or simply sin^2(theta/2). Your realistic requirement is M(xy) < M(xz) + M(yz) which is the same as saying: 0 < M(xz) + M(yz) - M(xy). Substituting the right hand side, you get something like:

M(xz)* + M(yz) - M(xy) =

sin^2(90/2) + sin^2(45/2) - sin^2(135/2) =

.5 + .1465 - .8535 =

-.2070**

Oops, this was supposed to be greater than zero per your realism requirement! So the realism requirement is flat out inconsistent with the predictions of QM. So here are the specific values that lead to a violation.

The question on the above ( as an outsider to QM ) I have is that you are applying QM predictions to negate the realism requirement. That would be like using realism predictions
to negate the QM requirement. That is why I asked what the actual data is that violates
the inequality. Having said that, I could give up realism in all these discussions for
saving locality. Especially when Bell/EPR experiments are done with photons.
 
  • #69
morrobay said:
The question on the above ( as an outsider to QM ) I have is that you are applying QM predictions to negate the realism requirement. That would be like using realism predictions to negate the QM requirement.

Having said that, I could give up realism in all these discussions for
saving locality.

That is what we are doing, and it makes perfect sense. QM is incompatible with local realism, predictions as defined above, and that was Bell's discovery. It is not often that such clear disagreements occur with such fundamental ideas.

Giving up realism for QM + locality is a good trade, in my opinion.
 
  • #70
As regards Bell's theorem, locality or localism refers to the particular form in which Bell has expressed it in his lhv model of quantum entanglement. Since that form is necessarily realistic (ie., expressed in terms of hidden variables), then BI violation can't entail the option of keeping either locality or realism in a model of quantum entanglement. As far as Bell's lhv formulation is concerned locality and realism are inseparable.
Keeping in mind that it's only locality and realism as formalized by Bell in his lhv model of quantum entanglement that are relevant.
 
<H2>1. Why are Bell's inequalities important in quantum mechanics?</H2><p>Bell's inequalities are important in quantum mechanics because they provide a way to test the validity of quantum theory against classical theories. The violation of these inequalities shows that quantum mechanics cannot be explained by classical theories and therefore highlights the unique nature of quantum systems.</p><H2>2. What is the significance of Bell's inequalities being violated?</H2><p>The violation of Bell's inequalities is significant because it demonstrates the existence of quantum entanglement, a phenomenon where two or more particles become connected in such a way that the state of one particle is dependent on the state of the other, regardless of the distance between them. This violates the principle of local realism and challenges our understanding of how the physical world operates.</p><H2>3. How do experiments show the violation of Bell's inequalities?</H2><p>Experiments designed to test Bell's inequalities involve measuring the correlation between the properties of two entangled particles. By comparing the results with the predictions of classical theories, researchers can determine if the inequalities are violated. Numerous experiments have been conducted, all of which have shown a violation of Bell's inequalities and therefore support the principles of quantum mechanics.</p><H2>4. Can Bell's inequalities be explained by hidden variables?</H2><p>No, Bell's inequalities cannot be explained by hidden variables. Hidden variables are theoretical properties that are not directly observable but are assumed to determine the outcome of experiments. However, the violation of Bell's inequalities has been confirmed through experiments, ruling out the possibility of hidden variables as an explanation.</p><H2>5. How do Bell's inequalities relate to the concept of non-locality?</H2><p>Bell's inequalities are closely related to the concept of non-locality, which refers to the ability of entangled particles to influence each other's properties instantaneously, regardless of the distance between them. The violation of Bell's inequalities is evidence of non-locality and challenges our understanding of causality in the physical world.</p>

1. Why are Bell's inequalities important in quantum mechanics?

Bell's inequalities are important in quantum mechanics because they provide a way to test the validity of quantum theory against classical theories. The violation of these inequalities shows that quantum mechanics cannot be explained by classical theories and therefore highlights the unique nature of quantum systems.

2. What is the significance of Bell's inequalities being violated?

The violation of Bell's inequalities is significant because it demonstrates the existence of quantum entanglement, a phenomenon where two or more particles become connected in such a way that the state of one particle is dependent on the state of the other, regardless of the distance between them. This violates the principle of local realism and challenges our understanding of how the physical world operates.

3. How do experiments show the violation of Bell's inequalities?

Experiments designed to test Bell's inequalities involve measuring the correlation between the properties of two entangled particles. By comparing the results with the predictions of classical theories, researchers can determine if the inequalities are violated. Numerous experiments have been conducted, all of which have shown a violation of Bell's inequalities and therefore support the principles of quantum mechanics.

4. Can Bell's inequalities be explained by hidden variables?

No, Bell's inequalities cannot be explained by hidden variables. Hidden variables are theoretical properties that are not directly observable but are assumed to determine the outcome of experiments. However, the violation of Bell's inequalities has been confirmed through experiments, ruling out the possibility of hidden variables as an explanation.

5. How do Bell's inequalities relate to the concept of non-locality?

Bell's inequalities are closely related to the concept of non-locality, which refers to the ability of entangled particles to influence each other's properties instantaneously, regardless of the distance between them. The violation of Bell's inequalities is evidence of non-locality and challenges our understanding of causality in the physical world.

Similar threads

Replies
4
Views
954
Replies
3
Views
772
Replies
71
Views
3K
Replies
72
Views
3K
Replies
0
Views
651
  • Quantum Physics
Replies
5
Views
1K
  • Quantum Physics
Replies
16
Views
2K
Replies
50
Views
4K
  • Quantum Physics
Replies
1
Views
699
Replies
25
Views
2K
Back
Top